Difference between revisions of "2018 AMC 10A Problems/Problem 12"
m (→Solution) |
(→Solution 2) |
||
Line 35: | Line 35: | ||
− | Case 1: <math>y>1</math> | + | <math>\textbf{Case 1:}</math> <math>y>1</math> |
− | <math>3-3y</math> will be negative so <math>|3-3y| = 3y-3</math> | + | <math>3-3y</math> will be negative so <math>|3-3y| = 3y-3.</math> |
<math>|3y-3-y| = |2y-3| = 1</math> | <math>|3y-3-y| = |2y-3| = 1</math> | ||
Line 46: | Line 46: | ||
− | Case 2: <math>y = 1 | + | <math>\textbf{Case 2:}</math> <math>y = 1 |
x = 0</math> | x = 0</math> | ||
− | Case 3: <math>y<1</math> | + | <math>\textbf{Case 3:}</math> <math>y<1</math> |
<math>3-3y</math> will be positive so <math>|3-3y-y| = |3-4y| = 1</math> | <math>3-3y</math> will be positive so <math>|3-3y-y| = |3-4y| = 1</math> | ||
Subcase 1: <math>y>4/3</math> | Subcase 1: <math>y>4/3</math> | ||
− | <math>3-4y</math> will be negative so <math>4y-3 = 1</math> | + | <math>3-4y</math> will be negative so <math>4y-3 = 1</math> \rightarrow <math>4y = 4</math>. There are no solutions (again, <math>y</math> can't equal to <math>1</math>) |
Subcase 2: y<4/3 | Subcase 2: y<4/3 | ||
− | <math>3-4y</math> will be positive so <math>3-4y = 1</math> | + | <math>3-4y</math> will be positive so <math>3-4y = 1</math> \rightarrow <math>4y = 2</math>. <math>y = \frac{1}{2}</math> and <math>x = \frac{3}{2}</math>. |
− | |||
− | |||
− | + | Thus, the solutions are: <math>(-3,2), (0,1), \left(\frac{3}{2},\frac{1}{2} \right)</math>, and the answer is <math>3,</math> or <math>\boxed{\textbf{(C)}}</math> | |
− | Solution by Danny Li JHS | + | Solution by Danny Li JHS, <math>\text{\LaTeX}</math> edit by pretzel. |
== See Also == | == See Also == |
Revision as of 22:41, 8 February 2018
Contents
Problem
How many ordered pairs of real numbers satisfy the following system of equations?
Solution
The graph looks something like this:
Now it's clear that there are intersection points. (pinetree1)
Solution 2
can be rewritten to . Substituting for in the second equation will give . Splitting this question into casework for the ranges of y will give us the total number of solutions.
will be negative so
Subcase 1:
is positive so and and
Subcase 2:
is negative so . and so there are no solutions ( can't equal to )
will be positive so
Subcase 1:
will be negative so \rightarrow . There are no solutions (again, can't equal to )
Subcase 2: y<4/3
will be positive so \rightarrow . and .
Thus, the solutions are: , and the answer is or
Solution by Danny Li JHS, edit by pretzel.
See Also
2018 AMC 10A (Problems • Answer Key • Resources) | ||
Preceded by Problem 11 |
Followed by Problem 13 | |
1 • 2 • 3 • 4 • 5 • 6 • 7 • 8 • 9 • 10 • 11 • 12 • 13 • 14 • 15 • 16 • 17 • 18 • 19 • 20 • 21 • 22 • 23 • 24 • 25 | ||
All AMC 10 Problems and Solutions |
2018 AMC 12A (Problems • Answer Key • Resources) | |
Preceded by Problem 9 |
Followed by Problem 11 |
1 • 2 • 3 • 4 • 5 • 6 • 7 • 8 • 9 • 10 • 11 • 12 • 13 • 14 • 15 • 16 • 17 • 18 • 19 • 20 • 21 • 22 • 23 • 24 • 25 | |
All AMC 12 Problems and Solutions |
The problems on this page are copyrighted by the Mathematical Association of America's American Mathematics Competitions.